The question stem asks: "The reasoning in the journalist's argument is most vulnerable to criticism on the grounds that the argument fails to consider that..." and the correct answer, E, says the flaw is that those who donate might not be those who join the party, making the necessary 30% benchmark of support unreachable.
However, this would then SUPPORT the conclusion of the journalist, who says that an educational party is unviable in the long-run.
So, is it then possible to support a conclusion, but criticize a stimulus for failing to do the best possible job of constructing its conclusion (i.e. here we criticize the argument, but not the conclusion)? If anything, this feels like an assumption question.
Hopefully my question makes sense.
Thanks!
https://classic.7sage.com/lsat_explanations/lsat-48-section-1-question-24/